LSAT and Law School Admissions Forum

Get expert LSAT preparation and law school admissions advice from PowerScore Test Preparation.

 Administrator
PowerScore Staff
  • PowerScore Staff
  • Posts: 8916
  • Joined: Feb 02, 2011
|
#35431
Complete Question Explanation

(See the complete passage discussion here: lsat/viewtopic.php?t=14270)

GR, Must. The correct answer choice is (D)

This is a fairly unique question for the reading comprehension section; the correct answer choice
will likely provide a scenario that would be much more likley to offend the standard of theoretical
equipoise than offend the standard of clinical equipoise.

Answer choice (A): This choice would be equally likely to violate either type of equipoise, so this
cannot be the right answer.

Answer choice (B): This choice would be unlikely to violate either theoretical or clinical equipoise,
so it should be ruled out of contention.

Answer choice (C): The scenario presented in this choice would be likely to violate either one of the
two types of equipoise discussed in the passage.

Answer choice (D): This is the correct answer choice, although such preferences would be sure to
offend the standard of theoretical equipoise (which requires a lack of any preference), the scenario
presented in this answer choice conforms perfectly to the standard of clinical equipoise as described
beginning on line 52. Preferences among participating physicians should not, argues the author, be
an ethical bar, as long as there is the noted recognition that their less favored treatment is preferred by a sizable constituency of experts within the medical field.

Answer choice (E): This scenario would not likely offend either theoretical or clinical equipoise, so
it cannot be the right answer to this Must Be True question.
 15veries
  • Posts: 113
  • Joined: Sep 25, 2016
|
#30455
What a brutal question...remind me of LR pattern question.
Anyways,
I was not sure about A.
"most" means not all right? Wouldn't that mean there's disagreement and then not violate clinical but theoretical only?
 Adam Tyson
PowerScore Staff
  • PowerScore Staff
  • Posts: 5153
  • Joined: Apr 14, 2011
|
#30556
"Most" means "more than half", 15, and that does include "all". In "real life", when we say most we typically mean more than half but less than all, and many people would interpret it to mean substantially more than half. The LSAT, however, exists in a parallel universe, similar to ours in most respects but differing in certain key ways, including in its rigorous application of logic. In that world, "most" can include "all", because that is more than half. "Most people need air and water to survive" is a true statement, because we all do.

Answer A would jeopardize both clinical and theoretical equipoise. Theoretical would be at risk because the researchers would have developed a clear preference for one treatment over the other ("strikingly favored"). Clinical would also be at risk because their might no longer be a sizable constituency within the medical profession as a whole that still favors the other treatment. There could be - maybe as many as half minus 1 person - but it does raise the possibility that the preference for the one treatment is overwhelming, maybe even a complete consensus. Again, "strikingly favored" suggests that there would no longer be that sizable constituency.

You're right that this was brutal, and it does indeed resemble a complex Parallel Reasoning question from LR. Good eye there! Just be sure to apply the same standards of rigor to questions like this one as to those in LR, and you'll be good to go.
 MikeRov25
  • Posts: 18
  • Joined: Jan 14, 2016
|
#30991
I understand how the first part of the answer "initial results..." violates the theoretical equipoise...but it is the latter portion "this does not affect their recognition..." that is confusing to me. How does that portion fit into the answer?
 Adam Tyson
PowerScore Staff
  • PowerScore Staff
  • Posts: 5153
  • Joined: Apr 14, 2011
|
#31088
That last part is what tells us that the situation described would not jeopardize clinical equipoise, Mike, and that's an important part of this question stem. We want to see a situation that jeopardizes theoretical, but not clinical, equipoise. As long as the researchers recognize that there is legitimate lack of consensus, they should be able to maintain clinical equipoise.

I hope that clears it up for you!
 swong1267
  • Posts: 24
  • Joined: Nov 25, 2017
|
#42808
Why is C wrong/how does it violate clinical equipose? Is it because there is consensus within the community?
 James Finch
PowerScore Staff
  • PowerScore Staff
  • Posts: 943
  • Joined: Sep 06, 2017
|
#42828
Hi SWong,

Yes, the last paragraph states that a necessary condition for clinical equipoise is a lack of consensus within the clinical community, with "a sizeable constituency" preferring other treatments; this alternative preference must in turn be recognized for clinical equipoise to exist. So if there is a consensus, clinical equipoise is already violated/moot.

Answer choice (D) works because it contains a lack of consensus and recognition of this lack of consensus on the part of researchers, which allows for clinical equipoise, while also violating theoretical equipoise by having some researchers with a preference for one treatment over others.

Hope this clears things up!
 Coleman
  • Posts: 44
  • Joined: Jul 07, 2020
|
#83163
Adam Tyson wrote: Thu Dec 01, 2016 2:47 pm That last part is what tells us that the situation described would not jeopardize clinical equipoise, Mike, and that's an important part of this question stem. We want to see a situation that jeopardizes theoretical, but not clinical, equipoise. As long as the researchers recognize that there is legitimate lack of consensus, they should be able to maintain clinical equipoise.

I hope that clears it up for you!
So what I understand is

"Initial results from the trial convince several of the participating physicians that one treatment more effectively treats the condition than the other does" - a lack of consensus is emerging in the post-trial phase since a group of physicians were convinced that one is more effective than the other, thus TE (theoretical equipoise) is violated

"This does not affect their recognition of the lack of consensus among experts in treating the disease" - These physicians think there are two groups that favor different treatments respectively, thus CE is maintained.

Is this reasoning correct? I found the second prong of the answer choice is kind of convoluted which is hard to discern what is its significance/implication. Any clarification will be greatly appreciated.

Thank you!
 Adam Tyson
PowerScore Staff
  • PowerScore Staff
  • Posts: 5153
  • Joined: Apr 14, 2011
|
#83305
Perfect, Coleman! Well done!

Get the most out of your LSAT Prep Plus subscription.

Analyze and track your performance with our Testing and Analytics Package.